Difference between revisions of "2002 AMC 12B Problems/Problem 9"

(Solution)
(See also)
Line 12: Line 12:
  
 
==See also==
 
==See also==
 +
{{AMC12 box|year=2002|ab=B|num-b=8|num-a=10}}
 +
 +
[[Category:Introductory Algebra Problems]]

Revision as of 09:49, 5 February 2008

Problem

If $a,b,c,d$ are positive real numbers such that $a,b,c,d$ form an increasing arithmetic sequence and $a,b,d$ form a geometric sequence, then $\frac ad$ is

$\mathrm{(A)}\ \frac 1{12} \qquad\mathrm{(B)}\ \frac 16 \qquad\mathrm{(C)}\ \frac 14 \qquad\mathrm{(D)}\ \frac 13 \qquad\mathrm{(E)}\ \frac 12$

Solution

We can let a=1, b=2, c=3, and d=4. $\frac{a}{d}=\frac{1}{4}  \Rightarrow \boxed{\mathrm{(C)}}$

See also

2002 AMC 12B (ProblemsAnswer KeyResources)
Preceded by
Problem 8
Followed by
Problem 10
1 2 3 4 5 6 7 8 9 10 11 12 13 14 15 16 17 18 19 20 21 22 23 24 25
All AMC 12 Problems and Solutions